- PowerScore Staff
- Posts: 5972
- Joined: Mar 25, 2011
- Sat Apr 08, 2017 11:59 am
#87691
Complete Question Explanation
(The complete setup for this game can be found here: lsat/viewtopic.php?t=1469)
The correct answer choice is (B)
This question asks you to identify the pair of courses in which at least one of the two courses must always be taken. Courses that have more restrictions—such as M—are not attractive candidates to appear in the correct answer because they are difficult to pair with other variables. If the pair listed must always have at least one course in the group of three or four courses taken, then variables with fewer restrictions are more likely candidates than variables with more restrictions. Thus, using that analytical principle, L and T is the most likely pairing for the correct answer because L and T are the two least restricted variables in the game.
Answer choice (B) contains L and T, and ultimately (B) is the correct answer. But, if we did not use the analytical pattern above, is there another way to arrive at this answer? Yes. One method is to use the prior hypotheticals to eliminate answer choices. Using this method, we can use the solution to question #18 (H, P, T) to eliminate answer choice (C). The solution to question #20 (L, S, T) eliminates answer choice (D). And the solution to question #21 (M, L, S/W) eliminates answer choice (A). Thus, using hypotheticals would eliminate every answer choice except (B) and (C). You could then make a hypothetical for answer choice (B) or (C) to determine which was correct (to make a hypothetical that proves or disproves an answer, attempt to make a viable solution that uses neither of the two courses in the answer. If a viable solution can be found, the answer is incorrect; if a viable solution cannot be found, the answer is correct).
(The complete setup for this game can be found here: lsat/viewtopic.php?t=1469)
The correct answer choice is (B)
This question asks you to identify the pair of courses in which at least one of the two courses must always be taken. Courses that have more restrictions—such as M—are not attractive candidates to appear in the correct answer because they are difficult to pair with other variables. If the pair listed must always have at least one course in the group of three or four courses taken, then variables with fewer restrictions are more likely candidates than variables with more restrictions. Thus, using that analytical principle, L and T is the most likely pairing for the correct answer because L and T are the two least restricted variables in the game.
Answer choice (B) contains L and T, and ultimately (B) is the correct answer. But, if we did not use the analytical pattern above, is there another way to arrive at this answer? Yes. One method is to use the prior hypotheticals to eliminate answer choices. Using this method, we can use the solution to question #18 (H, P, T) to eliminate answer choice (C). The solution to question #20 (L, S, T) eliminates answer choice (D). And the solution to question #21 (M, L, S/W) eliminates answer choice (A). Thus, using hypotheticals would eliminate every answer choice except (B) and (C). You could then make a hypothetical for answer choice (B) or (C) to determine which was correct (to make a hypothetical that proves or disproves an answer, attempt to make a viable solution that uses neither of the two courses in the answer. If a viable solution can be found, the answer is incorrect; if a viable solution cannot be found, the answer is correct).
Dave Killoran
PowerScore Test Preparation
Follow me on X/Twitter at http://twitter.com/DaveKilloran
My LSAT Articles: http://blog.powerscore.com/lsat/author/dave-killoran
PowerScore Podcast: http://www.powerscore.com/lsat/podcast/
PowerScore Test Preparation
Follow me on X/Twitter at http://twitter.com/DaveKilloran
My LSAT Articles: http://blog.powerscore.com/lsat/author/dave-killoran
PowerScore Podcast: http://www.powerscore.com/lsat/podcast/